You are on page 1of 9

POWER PLANT ENGINEERING MODULE I

A thermal power plant has a heat rate of 11,363 Btu/kw-hr. Find the thermal efficiency of the plant.
A. 28% *B. 30% C. 34% D. 40%
Solution:
eth = 3412/Heat rate = 3412/11,363 = 30%

Find the enthalpy of water at 212˚F and 14.7 psi if the dryness factor is 30%. Use the approximate
enthalpy formula of liquid.
A. 461 Btu/lb *B. 471 Btu/lb C. 481 Btu/lb D. 491 Btu/lb
Solution:
ht = (˚F – 32) = (212 – 32) = 180 Btu/lb
htg = 970 Btu/lb
h = ht + x htg
h = 180 + 0.3(970) = 471 Btu/lb

Find the enthalpy of Helium if its internal energy is 200 KJ/kg.


A. 144 KJ/kg B. 223.42 KJ/kg *C. 333.42 KJ/kg D. 168 KJ/kg
Solution:
R = 8.314/4 = 2.0785
K = 1.667 for helium
cp = k R/(k – 1) = 1.667(2.0785)/(1.667 – 1) = 5.195 KJ/kg-K
cv = R/(k – 1) = 2.0785/(1.667 – 1) = 3.116 KJ/kg-K
∆h/∆U = cp/cv
∆h/200 = 5.195/3.116
∆h = 333.42 KJ/kg

Convert 750˚R to ˚K
A. 390.33 ˚K B. 395.33 ˚K C. 410.33 ˚K *D. 416.33 ˚K
Solution:
˚R = 1.8 ˚K
750 = 1.8 ˚K
˚K = 416.667

An otto cycle has a compression ratio of 8. Find the pressure ratio during compression.
*A. 18.38 B. 16.38 C. 14.38 D. 12.38
Solution:
P1V1k = P2V22
(V1/V2)k = (P2/P1)
rkk = rp
rp = (8)1.4 = 18.38

A diesel cycle has a cut off ratio of 2.5 and expansion ratio of 4. Find the clearance of the cycle.
A. 9.11 % B. 5.55 % *C. 11.11 % D. 15.15 %
Solution:
rk = r c re
rk = 2.5(4) = 10
rk = (1 + c)/c
10 = (1 + c)/c
c = 11.11 %

A dual cycle has an initial temperature of 30 ˚C. The compression ratio is 6 and the heat addition at
constant volume process is 600 KJ/kg. If cut-off ratio is 2.5, find the maximum temperature of the cycle.
A. 3638.50 ˚C *B. 3365.50 ˚C C. 3565.50 ˚C D. 3965.50 ˚C
Solution:
T2 = T1 rkk-1 = (30 +273)(6)1.4-1 = 620.44 ˚K
QAV = m cv (T3 – T2)
600 = 1(0.7186)(T3 – 620.44)
T3 = 1455.396 ˚K
rc = T4/T3
2.5 = T4/1455.396
T4 = 3638.49 ˚K = 3365.50 ˚C

A 350 mm X 450 mm steam engine running at 280 rpm has an entrance steam condition of 2 Mpa and
230 oC and exit at 0.1 Mpa. The steam consumption is 2,000 kg/hr and mechanical efficiency is 85%. If
indicated mean effective pressure is 600 Kpa, determine brake thermal efficiency.
At 2 Mpa and 230 oC (Table 3): h1 = 2849.6 s1 = 6. 4423
At 0.1 Mpa: sf = 1.3026 hf = 417.46 sfg = 6.0568 hfg = 2258 hf2 = 417.46 KJ/kg
A. 23.34% *B. 15.25% C. 14.16% D. 27.34%
SOLUTION:
VD = 2[3.1416/4 (0.35) 2 (0.45)(280/60)]= 0.4041 m3/sec
Indicated Power = Pmi x VD = 600 x 0.4041 =242.45 KW
Brake Power = IP (em) = 242.45 (0.85) = 206.08KW
Brake Power 206.08 = 15.25%
etb = ms (h1-hf2) = (2000/3600)(2849.6 – 417.46)

A steam turbine receives 5,000 kg/hr of steam at 5 Mpa and 4000oc and velocity of 30 m/sec. It leaves the
turbine at 0.06 Mpa and 85% quality and velocity of 15 m/sec. Radiation loss is 10,000 KJ/hr. Find he
KW developed.
At 5 Mpa and 400oC: h1 = 3195.7 KJ/kg s1 =6.6459
At 0.006 Mpa: hf = 151.53 hfg = 2415.9
A. 1273.29 B. 2173.29 *C. 1373.60 D. 7231.29
SOLUTION:
h2 = hf + xhfg = 151.53 + 0.85(2415.9) = 2205.045 KJ/ kg
KE1 = ½ m v2 = ½ (5,000/3600)(30)2 = 625 W = 0.625 KW
KE2 = ½ m v2 = ½ (5,000/3600)(15)2 = 156.25 W = 0.15625 KW
By energy balance:
KE1 + mh1 = KE2 + mh2 + Q + W
W = (KE1 – KE2) + m(h1-h2) – Q
5000 10,000
W = (0.625 – 0.156) + (3600)(3195.7 – 2205.045) – 3600 = 1373.60 KW

A steam turbine with 85% stage efficiency receives steam at 7 Mpa and 550oC and exhausts as 20 Kpa.
Determine the turbine work.
At 7 Mpa and 550oC: h1 = 3530.9 Kj/kg s1 = 6.9486
At 20 Kpa (0.020 Mpa): sf = 0.8320 hf = 251.4 sfg = 7.0766 hfg = 2358.3
A. 1,117 KJ/kg B. 1,132 KJ/ kg C. 1,123.34 KJ/ kg *D. 1,054.95 KJ/kg
SOLUTION:
s1 = s2 = sf + xsfg
6.9486 = 0.8320 + x(7.0766)
x = 0.8643
h2 = 251.40 + 0.8643(2358.3) = 2289.78 KJ/kg
h1 - h2a
eST = h1 – h2
0.85 =3530.9 – h2a
3530.9 – 2289.78
h2a = 2475.95 KJ/kg
WT = h1 = h2a = 3530.9 – 2475.95 = 1,054.95 KJ/kg

A steam turbine with 80% stage efficiency receives steam at 7 Mpa and 550oC and exhaust as 20 Kpa.
Determine the quality at exhaust.
At 7 Mpa and 550oC: h1 = 3530.9 Kj/kg s1 = 6.9486
At 20 Kpa (0.020 Mpa): sf = 0.8320 hf = 251.4
*A. 96.96% B. 76.34% C. 82.34% D. 91.69%
SOLUTION:
sfg = 7.0766 hfg = 2358.3
s1 = s2 = sf + sfg
6.9486 = 0.8320 + x(7.0766)
x = 0.8643
h2 = 251.40 + 0.8643(2358.3) = 2289.78 KJ/kg
nST = h1 – h2a
h1 – h2
0.80 = 3530.9 – h2a
3530.9 – 2289.78
h2a = 2538.004 KJ/kg
h2a = hf + x hfg
2538.004 = 251.40 + x (2358.3)
x = 96.96%

A 16,000KW geothermal plant has a generator efficiency and turbine efficiency of 90% and 80%.,
respectively if the quality after throttling is 20% and each well discharges 400, 000 kg/hr, determine the
number of wells are required to produce if the charge of enthalpy if the change of enthalpy at entrance
and exit of turbine is 500KJ/kg.
A. 4 wells *B. 2 wells C. 6 wells D. 8 wells
SOLUTION:
WT = ms(h3 – h4)
16,000 = ms (500)
0.9(0.8)
ms = 44.44 kg/sec
ms = 160,000 kg/hr
160,000 = 0.20 mg
mg = 800,000 kg/hr
No. of wells = 800,000/400,000 = 2 wells

A liquid dominated geothermal plant with a single flash separator receives water at 204oC. The separator
pressure is 1.04 Mpa. A direct contact condenser operates at 0.034 Mpa. The turbine has a polytropic
efficiency of 0.75. For a cycle output of 60 MW, what is the mass flow rate of the well-water in kg/s?
At 204oC: hf = 870.51 KJ/kg
At 1.04 Mpa: hf = 770.38 hfg = 2009.2 hg = 2779.6 sg = 6.5729
At 0.034 MPa: hf = 301.40 hfg = 2328.8 sf = 0.9793 sfg = 6.7463
*A. 2,933 B. 2,100 C. 1,860 D. 2,444
SOLUTION:
h3 = hg at 1.04 MPa = 2779.6 KJ/kg
Solving for h4:
s3 = s4 = sf + xsfg
6.5729 = 0.9793 + x4(6.7463)
x4 = 0.829
h4 = 301.4 + 0.829(2328.8) = 2232.3 KJ/kg
WT = ms (h3 – h4)
60,000 = ms (2779.6 – 2232.3) 0.75
ms = 146.17 kg/sec
Solving for x2: (h1 = h2)
h1 = h2 = hf + xhfg
870.51 = 770.38 + x2(2009.2)
x2 = 0..049836
ms = x mg
146.17 = 0.049836 mg
mg = 2,933.06 kg/sec

An engine-generator rated 9000 KVA at 80% power factor, 3 phase, 4160 V has an efficiency of 90%. If
overall plant efficiency is 28%, what is the heat generated by the fuel.
A. 18,800 KW B. 28,800 KW C. 7500 KW *D. 25,714 KW
SOLUTION:
Gen. Output = pf x KVA = 0.8 x 9000 = 7200 KW
eoverall= Gen. Output
Qg

0.28 = 7200/Qg
Qg = 25,714.28 KW

The indicated thermal efficiency of a two stroke diesel engine is 60%. If friction power is 15% of heat
generated, determine the brake thermal efficiency of the engine.
A. 43% *B. 45 % C. 36% D. 37%
SOLUTION:
ne = IP/ Qg
0.60 = IP/Qg
IP = 0.60 Qg
BP = IP- FP = 0.60Qg – 0.15Qg = 0.45Qg
etb = BP/Qg = 0.45Qg/Qg = 45%

A 305 mm x 457 mm four stroke single acting diesel engine is rated at 150 KW at 260 rpm. Fuel
consumption at rated load is 0.56 kg/KW-hr with a heating value of 43,912 KJ/kg. Calculate brake
thermal efficiency
A. 10.53% B. 27.45% *C. 14.64% D. 18.23%
SOLUTION:
mf = 0.56 kg/KW-hr x 150 KW = 84 kg/hr = 0.0233 kg/sec
Brake Power 150
Brake thermal efficiency = = =14.64 %
mfQh 0.0233(43,912)

A waste heat recovery boiler produces 4.8 Mpa(dry saturated) steam from 104°C feedwater. The boiler
receives energy from 7 kg/sec of 954°C dry air. After passing through a waste heat boiler, the
temperature of the air is has been reduce to 343°C. How much steam in kg is produced per second? Note:
At 4.80 Mpa dry saturated, h = 2796.
A. 1.30 B. 0.92 *C. 1.81 D. 3.43
SOLUTION:
hf = approximate enthalpy of feedwater
hf = Cpt
hf = 4.187(104)
hf = 435.45 KJ/kg
Heat loss = Heat gain
m gc p(t 1 - t 2) = m s(h - h f)
7(1.0)(954 – 343) = ms(2796.0 – 436.45)
m s = 1.81 kg/sec

A diesel electric plant supplies energy for Meralco. During a 24-hour period, the plant consumed 240
gallons of fuel at 28°C and produced 3930 KW-hr. Industrial fuel used is 28°API and was purchased at
P30 per liter at 15.6°C. What is the cost of the fuel be to produce one KW-hr?
*A. P6.87 B. P1.10 C. P41.07 D. P5.00
SOLUTION:
SG 15.6C = 141.5/(131.5 + 28) = 0.887
Density at 15.6°C = 0.887(1kg/li) = 0.887 kg/li
SG 28C = 0.887[1-.0007(1 – 15.6)] = .879
Density at 28°C = 0.879(1 kg/li) = 0.879 kg/li
V28C / V15.6C = SG15.6C / SG28C
240 / V15.6C = 0.887 / 0.879
V15.6C = 237.835 gallons x 3.785 li/gal = 900.21 li
Cost = [(30)(900.21)] / 3930 = P6.87/KW-hr

In a gas turbine unit, air enters the combustion chamber at 550 kpa, 277°C and 43 m/s. The products of
combustion leave the combustor at 511 kpa, 1004°C and 180 m/s. Liquid fuel enters with a heating value
of 43,000 KJ/kg. For fuel-air ratio of 0.0229, what is the combustor efficiency of the unit in percent?
A. 70.38% B. 79.385% *C. 75.38% D. 82.38%
SOLUTION:
Heat supplied by fuel = mfQh = 0.0229(43,000) = 984.7 KJ/kg air
Q = heat absorbed by fuel
Q/m = Cp(T2 – T1) + ½(V22 – V12)
Q/m = (1.0)(1004 – 277) + ½[(180) 2 –(43) 2]/1000 =742.28 KJ/kg air
742.28
Combustor Efficiency = = 75.38%
984.7

The specific speed of turbine is 85 rpm and running at 450 rpm. If the head is 20 m and generator
efficiency is 90%, what is the maximum power delivered by the generator.
A. 450.51 KW B. 354.52 KW C. 650.53 KW *D. 835.57 KW
SOLUTION:
NS = (N√HP)/h5/4
85 = (450√HP)/(20 x 3.281) 5/4
Hp = 1244.52
Generator Output = (1244.52 x 0.746)(0.9) = 835.57 KW

In Francis turbine, the pressure gage leading to the turbine casing reads 380 Kpa. The velocity of water
entering the turbine is 8 m/sec, if net head of the turbine is 45 m, find the distance from center of spiral
casing to the tailrace.
*A. 3.0 m B. 3.5 m C. 4.0 m D. 4.5m

SOLUTION :

P
h= + z+ ¿ V2/2g
w
45 = (380/9.81) + z + [82/(2 x 9.81)]
z=3m

A turbine has a mechanical efficiency of 93%, volumetric efficiency of 95% and total efficiency of 82%. If
effective head is 40 m, find the total head.
A. 48.72 m B. 40.72 m *C. 36.22 m D. 34.72 m
SOLUTION:
eT = emehev
0.8 = 0.93(eh)(.95)
ηh = 0.9055
Total head = h eh = (40)(0.9055) = 36.22 m

A Pelton type turbine has 25 m head friction loss of 4.5 m. The coefficient of friction head loss (from
Moorse) is 0.00093 and penstock length of 80 m. What is the penstock diameter?
*A. 1,355.73 mm B. 3,476.12 mm C. 6771.23 mm D. 1686.73 mm
SOLUTION:
h =25- 4.5 = 20.5
v = √(2gh) = [(2 x 9.81 x 20.5)1/2] = 20.55 m/sec
hL = (2fLv2)/gD
4.5 = (2)(0.00093)(80)(20.055)2 / 9.81D
D = 1,355,730 m = 1,355.73 mm

In an 9,000 KW hydro-electric plant the over-all efficiency is 88% and the actual power received by the
customer is 110,000 KW-hrs for that day. What is the secondary power could this plant deliver during the
entire day?
A. 58,960 KW-hrs *B. 80,080 KW-hrs C. 65,960 KW-hrs D. 70,960 KW-hrs
SOLUTION:
Plant Capacity = 9,000(0.88)(24) = 190,080 KW-hrs
Secondary Power = 190,080 – 110,000 = 80,080 KW-hrs

A Pelton type turbine was installed 30 m below the gate of the penstock. The head loss due to friction is
12 percent of the given elevation. The length of penstock is 100 m and coefficient of friction is 0.00093.
Determine the power output in KW. ( Use Moorse equation)
A. 22,273 B. 23,234 C. 32,345 *D. 34,452
SOLUTION:
hL = 0.12(30) = 3.6 m
h = 30 – 3.6 = 26.40 m
v = (2gh)1/2 = [(2)(9.81)(26.4)]1/2 = 22.759 m/sec
hL= (2fLv2)/gD
3.6 = (2 x .00093 x 100 x 22.759) / (9.81D)
D = 2.728 m
π
Q = A x v = [ ( 2.758 )2](22.759) = 133.03 m3/sec
4
Power = w Q h = 9.81(133.03)(26.4) = 34,452 KW
A hydro electric plant having 30 sq. km reservoir area and 100 m head is used to generate power. The
energy utilized by the consumers whose load is connected to the power plant during a five-hour period is
13.5 x 106 kwh. The overall generation efficiency is 75%. Find the fall in the height of water in the
reservoir after the 5-hour period.
A. 5.13 m B. 1.32 m C. 3.21 *D. 2.20 m
SOLUTION
Energy Output = Power x time = (w Q h) x time
13.5 x 106 = 9.81(Q)(100)(0.75)(5)
Q = 3669.725 m3/s
Volume after 5 hrs = 3669.725(5 x 3600) = 66,055,050 m3
Volume = A x height
66,055,050 = (30 x 106) h
H =2.202 m

The gas density of chimney is 0.75 kg/m3 and air density of 1.15 kg/m3. Find the driving pressure if the
height of chimney is 63.71 m.
A. 0.15 kpa *B. 0.25 kpa C. 0.35 kpa D. 0.45 kpa
SOLUTION:
hw = H(da – dg) = 63.71(1.15 – 0.75) (0.00981) = 0.25 kpa

The actual velocity of gas entering in a chimney is 8 m/sec. The gas temperature is 25C with a gas
constant of 0.287 KJ/kg-K. Determine the gas pressure for a mass of gas is 50,000 kg/hr and chimney
diameter of 1.39m.
A. 95 kpa *B. 98 kpa C. 101 kpa D. 92 kpa
SOLUTION:
Vg = A x v = π / 4 (1.39)2(8) = 12.139 m3/s
PgVg = mgRgTg
P(12.139) = (50,000/3600)(.287)(25 +273)
P = 97.85 kpa

A steam generator with economizer and air heater has an overall draft loss of 25.78 cm of water. If the
stack gases are at 177C and if the atmosphere is at 101.3 Kpa and 26C, what theoretical height of stack
in meters is needed when no draft fan are used? Assume that the gas constant for the flue gases is the
same as that for air.
A 611.10 B. 631.10 *C.651.10 D.671.10
SOLUTION:
w = P/RT
da = (101.325)/[(.287)(26 + 273)] = 1.180 kg/m3
dg = (101.3)/[(0.287)(177 +273)] = 0.784 kg/m3
Draft = (0.2578)(1000) = 257.80 kg/m3
Draft = H(da – dw)
257.80 = H(1.18 – 0.784)
H = 651.10 m

A foundation measures 12 ft x 14 ft x16 ft. Find the number of sacks of cement needed for 1:2:4 mixture.
A. 302 B. 404 C. 356 *D. 598
SOLUTION:
V = 12 X 14 X 16 = 2,688 ft3 (1 yd3 / 33 ft3) = 99.55 yd3 of concrete
For every 1 yd3 concrete, it needs 6 sacks of cement
Therefore:
No. of sacks = 6(99.55) = 597.33 sacks or 598 sacks

A rectangular foundation cross-section has a bed plate dimension of 8 ft x 10 ft. The uniform clearance
on each side is 1 ft. The height of foundation is 4.5 ft. If the weight of the steel bar reinforcements needed
is 1/2% of weight of foundation, find the weight of steel bars. Use concrete density of 2400 kg/m3 .
A. 173.47 kg *B. 183.47 kg C. 163.47 kg D. 153.47 kg
SOLUTION:
A = (8 + 2) (10 + 2) = 120 m2
V = Ah = 120(4.5) = 540 ft3 = 15.29 m3
W = wV = (2400)(15.29) = 36,693.25 kg
Weight of steel bars = (1/2%) Wf = 0.005(36,693.25) = 183.47 kg

A steam pipe having a surface temperature of 250C passes through a room where the temperature is 27
C. The outside diameter of pipe is 100 mm and emissivity factor is 0.8. Calculate the radial heat loss for 3
m pipe length.
A. 1434.7 W B. 37.46 W *C. 2651.82 W D. 3545.45 W
SOLUTION:
A = πDL = π ( 0.10 ) (3) = 0.425m2
Solving for heat due to radiation:
Tg = 250 +273 = 523K
T2 = 27 +273 = 300K
Qa = 20,408.4 x 104 AF(T14 – T24), J/hr = 20,408.4 x 104(0.8)(0.7539)[(523)4 – (300)4]
Qr = 10,266,539.06 j/hr x 1hr/3600sec = 2851.82 W

A turbo-charged, 16 cylinder, Vee-type diesel engine has an air consumption of 3,000 kg/hr per cylinder
at rate load and speed. This air is drawn in through a filter by a centrifugal compressor directly
connected to the exhaust gas turine. The temperature of the air from the compressor is 135C and a
counter flow air cooler reduces the air temperature to 45C before it goes to the engine suction heater.
Cooling water enters air cooler at 30C and leaves at 40C. Calculate the log mean temperature difference.
A. 47.23C B. 87.82C *C. 43.34C D. 65.24C
SOLUTION:
∆ t a = 45-30 = 15C
∆ t b = 135 – 40 = 95C
∆ t mean = [∆ t a - ∆ t b] / [ln(∆ t a¿ ∆ t b)] = [95-15] / ln(95/15) = 43.34C

Water is flowing in a pipe with radius of 30 cm at a velocity of 5 m/s at the temperature in pipe. The
density and viscosity of the water are as follows: density 997.9 kg/sec viscosity = 1.131 Pa-s. What is the
Reynolds Number for this situation?
*A. 2647 B. 96.2 C. 3100 D. 1140
SOLUTION:
n = Dvg / v
Where:
D = 2(0.30) = 0.60 m
vg = 5 M/SEC
v = 1.131/997.9 = 0.0011334 m2 / sec
Nm = 0.60(5)/0.0011334 = 2,647

Compute the amount of condensate form during 10 minutes warm-up of 180 meter pipe conveys the
saturated steam with enthalpy vaporization hfg = 1,947.8 LJ/kg. The minimum external temperature of
pipe is 2C. The final temperature of pipe is 195C. The specific heat of pipe material is 0.6 KJ/kg-C. The
specific weight is 28 kg/m.
A. 249.69 kg B. 982.45 kg *C. 299.64 kg D. 423.45 kg
SOLUTION:
mp = mass of pipe = 28(180) = 5,040 kg
Heat Loss by steam = Heat loss from pipe
m(hg - hf) = mpcp (t2 – t1)
m(1947.8) = (5040)(0.6)(195-2)
m = 299.64 kg

The discharge pressure of an air compressor is 5 times the suction pressure. If volume flow at suction is
0.1 m3/sec, what is the suction pressure if compressor work is 19.57 kw? (use n=1.35)
A. 97 kpa *B.98 kpa C. 99 kpa D.100 kpa
SOLUTION:
nP 1V 1
W= [(P2/P1)n-1/n – 1]
n−1
19.57 = 1.35(P1)(0.1)/(1.35-1)[(5)1.35-1/1.35 – 1]
P1 = 98 KPa

The gain of entropy during isothermal nonflow process of 5 lb of air at 60℉ is 0.462 Btu/R. Find the
V1/V2.
A. 3.85 *B. 0.259 C. 1.0 D. 0.296
SOLUTION:
∆s = m R T ln(V2/V1)
0.462 = 5 (53.33/778) ln (V2/V1)
V2/V1 = 3.85
V1/V2 = 1/3.85 = 0.259

A plate – type solar energy collector with an absorbing surface covered by a glass plate is to receive an
incident radiation of 800 W/m2. The glass plate has a reflective of 0.12 and a transmissivity of 0.85. The
absorbing surface has an absorptivity of 0.90. The area of the collector is 5m2. How much solar energy in
watts is absorbing by the collector?
A. 2500 B. 2880 C. 3510 *D. 3060
SOLUTION:
Q = heat absorbed from sun
Q = (800 W/m2)(5 m2)(0.85)(0.9) = 3,060 watts

A simple Rankine cycle produces 40 MW of power, 50 MW of process heated and rejects 50 MW of heat
to the surroundings. What is the utilization factor of this cogeneration cycle neglecting the pump work?
A. 50% B. 60% *C. 64% D. 80%
SOLUTION:
QA = WT + WprocessWp = 40 + 50 + 50 = 140 KW
Q process +W T 50+ 40
UF = = = 64 %
QA 140

A heat engine receives heat from a source at 1200oK at a rate of 500KJ/s and rejects the waste heat to a
sink at 300oK. If the power output of the engine is 200KW, the second law efficiency of the heat engine is:
A.35% B.40% *C.53% D.75%
Solution:
ea= 200/500 = 0.40
et= (TH – TL)/TH = (1200 – 300)/1200 = 0.75
es= 0.40/0.75= 53.33%

By superheating the steam to a higher temperature in Rankine cycle, which of the following will
increase? (Use per unit analysis)
A. moisture content at the turbine exhaust B. pump work
C. condenser pressure D. cycle efficiency

By reheating the steam before entering the second stage in Rankine cycle, which of the following will
decrease?
A. Turbine work B. moisture content after expansion
C. heat added D. heat rejected

Economizer in a water tube boiler is heated by:


A. electric furnace B. electric current C. incoming flue gas D. outgoing flue gas

Sum of the maximum demand over the simultaneous maximum demand?


A. use factor B. capacity factor C. demand factor D. diversity factor

Regenerative with feed heating cycle with infinite number of feed water heaters thus efficiency is equal
to?
A. otto cycle B. stirling cycle C. erricson cycle D. carnot cycle

Transfers heat directly to electrical energy by utilizing thermionic emissions.


A. thermionic motor C. thermionic converter
B. thermionic generator D. thermionic cell

Is the largest group of coal containing 46-89% of fixed carbon and 20% to 40% volatile matter.
A. anthracite B. sub-anthracite C. bituminous D. sub-bituminous

When 1 gram of coal is subjected to a temperature of about 105C for a period of 1 hour, the loss in
weight of the sample gives the:
A. Volatile matter B. ash C. fixed carbon D. moisture content

You might also like